SlideShare a Scribd company logo
1 of 27
Download to read offline
Question # 101 (Multiple Choice) You are called for an assault victim. You arrive to find a man
approximately 20 years of age lying on his side in an alley. Examination reveals the patient has blood
oozing from the right side of his nose. The patient's right cheek appears flattened and there is discoloration
around his eyes. The patient states his cheek and nose feel numb. You suspect:
A) mandibular fracture
B) basilar skull fracture
C) parotid fracture
D) zygomatic fracture
Question # 102 (Multiple Choice) You are on the scene of a car-pedestrian crash. A 63-year-old woman
was crossing the street when she was struck by a car traveling approximately 45 miles per hour. The patient
was thrown approximately 20 feet. The decision to "load-and-go" or continue stabilization at the scene will
be primarily based on:
A) local triage protocols
B) national trauma guidelines.
C) employer policies & physical findings
D) mechanism of injury & physical findings.
Question # 103 (Multiple Choice) Select the correct statement regarding an epidural hematoma.
A) In an epidural hematoma, bleeding occurs between the dura and arachnoid mater.
B) An epidural hematoma develops slowly, producing symptoms hours or days after the
injury.
C) In an epidural hematoma, bleeding occurs between the cranium and the dura
D) An epidural hematoma is most often associated with venous bleeding
Question # 104 (Multiple Choice) A "Halo" test is a check for
A) cervical injury
B) CSF leaks
C) Priapism
D) Brain damage
Question # 105 (Multiple Choice) The METTAG system is one method of triaging patients. When using
this system, a black tag identifies a patient who is:
A) dead or unexpected survival.
B) non-life or limb-threatening.
C) less critically injured.
D) the most critically injured.
Question # 106 (Multiple Choice) Responsibilities of the incident commander include:
A) working with the communication center and hospitals to obtain medical facility status and
treatment capabilities.
B) evaluating the resources required for patient treatment and providing suitable "immediate"
and "delayed" treatment areas.
C) determining the resources necessary to extricate trapped patients and delivering them to
the treatment sector.
D) rapidly assessing the situation, requesting additional resources as necessary, and
developing a plan of action for management of the incident.
Question # 107 (Multiple Choice) The START method of triage uses a 60-second assessment that
evaluates:
A) pulses, motor, sensation
B) airway, breathing, circulation
C) ventilation, perfusion, mental status
D) level of consciousness, tenderness, crepitation
Question # 108 (Multiple Choice) The METTAG system is one method of triaging patients. When using
this system, a green tag identifies a patient who is:
A) dead or unexpected survival.
B) non-life or limb-threatening.
C) less critically injured.
D) the most critically injured.
Question # 109 (Multiple Choice) The START method of triage classifies victims as:
A) immediate, delayed, nonsalvageable/dead
B) delayed, non-life or limb-threatening, dead
C) the most critically injured, walking wounded, dead
D) walking wounded, nonsalvageable, the most critically injured
Question # 110 (Multiple Choice) The sector responsible for directing emergency vehicles at the scene of a
major incident is the:
A) staging sector
B) treatment sector
C) extrication sector
D) transportation sector
Question # 111 (Multiple Choice) A 27-year-old man has suffered a blow to the right side of the head. His
injury may cause compression of the oculomotor cranial nerve resulting in:
A) dilation of the ipsilateral pupil.
B) dilation of the contralateral pupil.
C) constriction of the ipsilateral pupil.
D) constriction of the contralateral pupil.
Question # 112 (Multiple Choice) You are called for an unconscious man. On arrival you find an
unresponsive male, approximately 20 years of age, lying supine in a parking lot. Bystanders state he was the
victim of a gang fight and has been struck in the head and shoulders with a baseball bat. Examination
reveals contusions to the left temporal area and over the shoulders. There is a small amount of bleeding
from the left nostril. The patient's blood pressure is 118/74, pulse 90 and regular, respiratory rate 12/min.
Which of the following represents the MOST IMPORTANT intervention you can perform for this patient to
help minimize cerebral edema?
A) Hyperventilate the patient.
B) Apply the pneumatic antishock garment.
C) Placing the patient in Trendelenburg position.
D) Establish two large-bore IVs of normal saline and infuse wide open.
Question # 113 (Multiple Choice) The Glasgow Coma Scale evaluates:
A) motor response, gag reflex, verbal response.
B) eye opening, motor response, verbal response.
C) eye opening, pupillary response, motor response.
D) verbal response, pupillary response, motor response.
Question # 114 (Multiple Choice) A patient who opens his eyes in response to pain, makes no verbal
response, but withdraws from pain has a Glasgow Coma Score of:
A) 3
B) 5
C) 7
D) 11
Question # 115 (Multiple Choice) Cushing's triad is associated with
A) cardiac tamponade
B) tension pneumothorax
C) massive hemothorax
D) increased intracranial pressure
Question # 116 (Multiple Choice) Which of the following signs are associated with Cushing's triad?
A) decreasing blood pressure, decreasing pulse rate, change in respiratory pattern
B) increasing blood pressure, decreasing pulse rate, change in respiratory pattern
C) increasing blood pressure, increasing pulse rate, change in respiratory pattern
D) decreasing blood pressure, increasing pulse rate, change in respiratory pattern
Question # 117 (Multiple Choice) Select the most appropriate statement that should be used to describe a
patient's level of consciousness
A) The patient is obtunded
B) The patient appears drowsy
C) The patient is semiconscious
D) The patient withdraws from a painful stimulus
Question # 118 (Multiple Choice) All of the following patients are victims involved in a motor vehicle
crash. Which patient should be rapidly extricated?
A) 26-year-old female front seat passenger complaining of a headache. Laceration present on
right forehead. BP 138/80, P 92, R 16
B) 19-year-old male back seat passenger complaining of pain in right femur. Swelling and
discoloration noted over right thigh. BP 128/74, P 88, R 14
C) 30-year-old male driver complaining of difficulty breathing. Unable to palpate radial
pulse, carotid weak, rate 120. Respiratory rate 44 and labored
D) 24-year-old female back seat passenger complaining of abdominal pain. Seatbelt markings
noted over chest and abdomen. BP 118/82, P 76, R 16
Question # 119 (Multiple Choice) The MOST important sign in the evaluation of a head-injured patient is:
A) tachycardia
B) blood pressure
C) tachypnea
D) level of consciousness
Question # 120 (Multiple Choice) LORAZAPAM (ATIVAN) is classified as an
A) benzodiazepine
B) vasodilator
C) anit-inflammatory agent
D) anticholinergic
Question # 121 (Multiple Choice) The patient in the "I Feel So Ejected" case was given ATIVAN at the
hospital for
A) reducing brain swelling
B) reducing hypotenison
C) reducing seizure activity
D) reducing PVC's
Question # 122 (Multiple Choice) The correct adult dosage of ATIVAN is
A) 0.5 - 2.0 mg IV
B) 1-2 g diluted in 10 ml of D5W
C) 2-4 g IV
D) 1.5 -2.0 g/kg IV
Question # 123 (Multiple Choice) The Mannitol dose for an adult is
A) 1.5 -2.0 g/kg IV
B) 25-50 mg IV
C) 0.5 mg/lb to 0.8 mg/lb IM, SQ
D) 0.2 ml in 2.5 ml saline
Question # 124 (Multiple Choice) MANNITOL (OSMOTROL) is an osmotic diuretic that inhibits sodium
and water absorption in the kidneys. It would most likely be used for
A) GI bleeds
B) acute cerebral edema
C) kidney injury
D) Loop of Henley dysfunction
Question # 125 (Multiple Choice) Select the correct statement regarding an epidural hematoma.
A) In an epidural hematoma, bleeding occurs between the dura and arachnoid mater.
B) n epidural hematoma develops slowly, producing symptoms hours or days after the injury.
C) In an epidural hematoma, bleeding occurs between the cranium and the dura.
D) An epidural hematoma is most often associated with venous bleeding.
Question # 126 (Multiple Choice) Signs and symptoms of a basilar skull fracture may include: 1.
Cerebrospinal fluid draining from the nose. 2. Ecchymosis in the mastoid region. 3. Periorbital ecchymosis.
A) a. 1, 2 only.
B) b. 1, 3 only.
C) c. 2, 3 only.
D) d. 1, 2, 3.
Question # 127 (Multiple Choice) Nitroglycerin may not be given if the patient
A) has used two tablets of nitroglycerin
B) has a systolic blood pressure less than 90
C) has a diastolic blood pressure less than 90
D) has taken one tablet with no relief
Question # 128 (Multiple Choice) OPQRST are letters used to help you remember which questions to ask
patients during your assessment. What does the "P" stand for?
A) proximity
B) pulse
C) pain
D) provocation
Question # 129 (Multiple Choice) A 45-year-old patient complains of breathing difficulty and and chest
pain. He will not tolerate a non-rebreather mask. You should
A) use a nasal cannula
B) move the mask to a position of comfort
C) provide artificial ventilation with a bag-valve mask
D) use a pocket mask with supplemental oxygen
Question # 130 (Multiple Choice) Your 40-year-old male patient complains of mild chest discomfort. As an
EMT-B you should:
A) decide whether or not the patient has a heart problem
B) decide what type of heart problem it might be
C) treat as if the patient has cardiac compromise.
D) all of the above
Question # 131 (Multiple Choice) Cardiac compromise commonly includes all of the following signs or
symptoms EXCEPT
A) difficulty breathing.
B) warm, dry skin.
C) nausea or vomiting.
D) epigastric pain
Question # 132 (Multiple Choice) Your adult patient with chest pain has a blood pressure of 92/54, a pulse
of 112 and has a history of angina. You should
A) transport the patient promptly.
B) administer the patient's prescribed nitroglycerin
C) quickly begin CPR/full resuscitation efforts
D) apply an automated external defibrillator and monitor the patient's cardiac rhythm
Question # 133 (Multiple Choice) Which of the following is a shockable rythum?
A) sinus rhythm
B) ventricular fibrillation
C) pulseless electrical activity
D) third degree block
Question # 134 (Multiple Choice) Which of the following cardiac arrest patients should you defibrillate?
A) a 12-year-old who weighs less 90 lbs
B) a 14-year-old who weighs 110 lbs
C) a 12-year-old trauma patient who weighs 120 lbs
D) an infant experiencing sudden infant death syndrome
Question # 135 (Multiple Answer) All of the following are effects of nitroglycerin (multi answer)
A) it dilates the blood vessels
B) it allows more blood to remain in the capillaries
C) it reduces the heart's work load
D) it increases the blood pressure
Question # 136 (Multiple Answer) Which of the following are contraindications for the use of
nitroglycerin? (multi answer)
A) The patient has a history of cardiac problems
B) The patient complains of chest pain
C) The patient has also has a head injury
D) The patient took Viagra 3 hours ago
Question # 137 (Multiple Choice) The cardiac conduction system disturbance that most commonly results
in cardiac arrest is
A) pulseless electrical activity
B) ventricular fibrillation
C) ventricular tachycardia
D) asystole.
Question # 138 (Multiple Choice) The EMT-B requests prehospital advanced cardiac life support for the
care of the cardiac arrest patient because
A) early ACLS intervention provides for higher survival rates
B) ACLS personnel must be present when EMT-Bs perform defibrillation.
C) only paramedics can transport cardiac arrest patients
D) the EMT-B is not adequately trained to manage cardiac arrest
Question # 139 (Multiple Choice) Nitroglycerin is a medication that is administered:
A) as a slurry
B) as an injection
C) sublingually.
D) by inhalation
Question # 140 (Multiple Choice) Why is nitroglycerin effective for patients with recurrent chest pain?
A) It enlarges bronchial tubes
B) It constricts the blood vessels
C) It dilates coronary blood vessels
D) All of the above
Question # 141 (Multiple Choice) Your 55-year-old female patient has chest pain and a blood pressure of
110/80. You give nitroglycerin. You recheck her blood pressure and are unable to obtain a reading. Her
response to the nitroglycerin is called:
A) a side effect
B) an indication
C) a contraindication
D) a characteristic
Question # 142 (Multiple Choice) Your 55-year-old female patient has chest pain and a blood pressure of
102/70. You assist with one tablet of nitroglycerin. You recheck her blood pressure and it is 80/50 and
become semi- consciousness. A an EMT B, you should
A) place her in the Fowlers position
B) place her in the semi reverse Trendelenburg position
C) place her in the semi Fowlers position
D) place her in the Trendelenburg position
Question # 143 (Multiple Choice) Study the picture, and then identify: Structure number 1
A) Inferior vena cava
B) Superior vena cava
C) Aorta
D) Right ventricle
Question # 144 (Multiple Choice) Study the picture, and then identify: Structure number 5
A) Right ventricle
B) Aorta
C) Inferior vena cava
D) Left ventricle
Question # 145 (Multiple Choice) Using the picture, which number indicates- The first chamber where
blood returns to the heart
A) 1
B) 2
C) 5
D) none of the above
Question # 146 (Multiple Choice) If there are no contraindications, ___ mg of ASA should be given to the
chest pain patient.
A) 362
B) 324
C) 400
D) 320
Question # 147 (Multiple Choice) The ST segment in lead II and II is
A) depressed
B) elevated
C) normal
D) isoelectric
Question # 148 (Multiple Choice) A lateral left ventricular myocardial infarction is due to the occlusion of:
A) Perforating arteries
B) Right coronary artery
C) Right marginal branch
D) Left anterior descending
E) Left circumflex artery
Question # 149 (Multiple Choice) Whichbest refers to the filling and emptying of the heart:
A) Blood Brain Barrier
B) Starlings Law
C) Einthoven's Triangle
D) Angle of Louis
E) Poiseulle's Law
Question # 150 (Multiple Choice) The formation of a thrombus
A) is the result of rupture of the artery wall
B) a thinning of the artery wall
C) is the result of myocardial cells becoming ischemic
D) can lead to an embolism obstructing blood flow in the artery
Question # 151 (Multiple Answer) The left coronary artery supplies (multi answer)
A) LEFT VENTRICLE
B) ANTERIOR 2/3 OF THE INTERVENTRICULAR SEPTUM
C) VARIABLE PORTIONS OF THE RIGHT VENTRICLE
D) ATRIOVENTRICULAR NODE
Question # 152 (Multiple Answer) The right coronary artery supplies (multi answer)
A) POSTEROLATERAL SURFACE OF THE HEART
B) SINOATRIAL NODE
C) POSTERIOR 1/3 OF THE INTERVENTRICULAR SEPTUM
D) LEFT VENTRICLE
Question # 153 (Multiple Choice) A myocardial infarction occurs when
A) a heart valve does not close completely
B) a heart valve does not open fully
C) there is a blockage in a coronary artery
D) the electrical impulses of the heart become irregular
Question # 154 (Multiple Choice) All of the following are complications of myocardial infarction except
A) Cor pulmonale
B) Cerebral emboli
C) Cardiac tamponade
D) Dissecting aortic aneurysm
Question # 155 (Multiple Choice) The reciprocal ST that matches the previous answer is seen in leads
A) aVL and V2
B) I, aVL and V2
C) aVL, V5 and V6
D) aVF, aVL and V2
Question # 156 (Multiple Choice) Deep and slurred S waves can be seen in leads
A) I, aVL, V5 and V6
B) V1 to V4
C) I, aVL and V2
Question # 157 (Multiple Choice) Poor R wave progression in the precordial leads along with ST elevation
in V1 could suggest
A) the presence of a septal infarction
B) the presence of an anterior infarction
C) the presence of an aortic tear
D) the presence of Cor pulmonale
Question # 158 (Multiple Choice) Patients with poor cardiac output post nitrogylcerin administration may
need to be treated with an inotropic drug, such as
A) lidocaine
B) atropine
C) dobutamine
D) mag sulfate
Question # 159 (Multiple Choice) Patients with poor cardiac output post nitrogylcerin administration may
need to be treated for symptomatic bradycardia with
A) lidocaine
B) atropine
C) dobutamine
D) mag sulfate
Question # 160 (Multiple Choice) BONUS QUESTION: Thyroid hyperfunction leads to:
A) Mitral stenosis
B) Marked tachycardia
C) Decreased cardiac output
D) Right-sided heart failure
E) Increased peripheral vascular resistance
Question # 161 (Multiple Choice) 1. The currently recognized cause for cat scratch disease is:
A) prion
B) rickettsia
C) fungus
D) virus
Question # 162 (True/False) 2. True or false, Cat Scratch disease is more common patients under the age of
21
A) True
B) False
Question # 163 (True/False) 3. Lymphadenopathy in the adult is frequently a sign of metastatic cancer
A) True
B) False
Question # 164 (Multiple Choice) Cats
A) Tularemia
B) Anthrax
C) Plague
D) Brucellosis
E) Toxiplasmosis
Question # 165 (Multiple Choice) Rats
A) Tularemia
B) Anthrax
C) Plague
D) Brucellosis
E) Toxiplasmosis
Question # 166 (Multiple Choice) Cattle
A) Tularemia
B) Anthrax
C) Plague
D) Brucellosis
E) Toxiplasmosis
Question # 167 (Multiple Choice) Sheep
A) Tularemia
B) Anthrax
C) Plague
D) Brucellosis
E) Toxiplasmosis
Question # 168 (True/False) 5. Lymphadenopathy in children is most commonly associated with infection
A) True
B) False
Question # 169 (True/False) 6. Declawing of cats is a proper prophylaxis for CSD
A) True
B) False
Question # 170 (True/False) 7. Humans may acquire CSD directly via bites from infected fleas.
A) True
B) False
Question # 171 (Multiple Choice) 8. Using the pneumonic AEIOU TIPS as a guide for evaluation of coma,
what does the letter A stand for?
A) Acidosis
B) Alkalosis
C) Akestenia
D) Amyloidosis
Question # 172 (Multiple Choice) 9. Causes of seizure GENERALLY include all of the following,
EXCEPT:
A) hypoxia
B) hypoglycemia
C) infection
D) pre-eclampsia
Question # 173 (True/False) 10. Cat bites are more commonly the cause of CSD than scratches
A) True
B) False
Question # 174 (Multiple Choice) A 54-year-old male patient has been thrown through a window. Glass has
caused an open neck wound. You should dress this wound with:
A) an occlusive dressing
B) a sterile gauze dressing
C) a moist gauze dressing
D) butterfly bandages
Question # 175 (Multiple Choice) Your patient has been exposed to an unknown chemical which has
soaked his clothing. You should flush the patient with
A) water
B) a neutralizing solution
C) vinegar
D) a baking soda solution
Question # 176 (Multiple Choice) Shock absorption and insulation are the major functions of which layer of
the skin?
A) dermis
B) subcutaneous layer
C) epidermis
D) dura.
Question # 177 (Multiple Choice) The major problem caused by electrical shock is often not the burn itself.
The most serious problem to consider is
A) shock
B) hypothermia
C) cardiac arrest
D) brain damage
Question # 178 (Multiple Choice) A 23-year-old male patient has been involved in a knife fight and has an
open abdominal injury with protruding bowel. Your treatment of the injury should include
A) replacing the exposed bowel
B) application of the PASG for the abdominal injury
C) covering the exposed bowel with a sterile dressing moistened with saline, then an
occlusive dressing
D) covering the bowel with dry sterile gauze prior to transport
Question # 179 (Multiple Choice) Because it can become a constricting band,______ should NOT use to
hold dressings in place.
A) a triangular bandage
B) an elastic bandage
C) strips of adhesive tape
D) self-adherent roller gauze
Question # 180 (Multiple Choice) A 4-year-old male patient was running in the school hall. You find a
pencil impaled in his upper arm. Your should
A) apply a tourniquet to control bleeding
B) remove the pencil and bandage the wound
C) cut the exposed portion of the pencil off to ease bandaging and transport
D) stabilize the pencil with a bulky dressing
Question # 181 (Multiple Choice) A 32-year-old female patient has amputated her index finger in a table
saw accident. You have bandaged her hand. The best way to transport the amputated finger is by
A) wrapping the finger in gauze, placing it in a plastic bag, and keeping it cool
B) placing the finger directly in a container sterile saline
C) wrapping the finger well in an occlusive dressing
D) placing the finger directly on dry ice
Question # 182 (Multiple Choice) A patient has had his lower legs burned. The area is charred black but
your patient states he has little pain. This burn would be classified as
A) minor
B) partial thickness
C) superficial
D) full thickness
Question # 183 (Multiple Choice) Acceptable care for a superficial burn may include all of the following
EXCEPT
A) apply a burn ointment
B) cool the burn with water
C) keep the patient warm
D) keep the burn site clean
Question # 184 (Multiple Choice) When force is transmitted to the body's internal structures causing
internal organs to rupture or bleed internally, or small bone fractures to occur is called a(n):
A) crush injury
B) concussion
C) avulsion
D) contusion.
Question # 185 (Multiple Choice) A bruise may be an indication of internal injuries and internal bleeding. It
is important for you to monitor your patient for any signs and symptoms of
A) infection
B) shock
C) low blood sugar
D) d. hypothermia
Question # 186 (Multiple Choice) Formation of blood clots would be this mans job :-)
A) White Knight Luke
B) Plumber Platelet
C) Erythrocyte Erythmic
D) Plasma Pooler
Question # 187 (Multiple Choice) If a patient is in shock, you would expect the skin to be
A) cool and clammy
B) warm and dry
C) hot and dry
D) none of the above
Question # 188 (Multiple Choice) What does the "D" in "DCAP-BTLS" stand for?
A) dislocations
B) distal function
C) dyspnea
D) deformities
Question # 189 (Multiple Choice) A teen male patient has a gunshot wound to the chest. Your detailed
physical exam on this patient
A) will begin immediately after the initial assessment
B) will begin before the initial assessment
C) may never begin due to the seriousness of the chest injury
D) will be delayed until an ALS provider is on the scene
Question # 190 (Multiple Choice) External bleeding that is a steady flow of dark red blood is from a(n):
A) artery
B) vein
C) capillary
D) all of the above
Question # 191 (Multiple Choice) A late sign of internal bleeding is:
A) bruising
B) painful, swollen extremities
C) signs of shock
D) bright red vomitus
Question # 192 (Multiple Choice) Early signs and symptoms of hypoperfusion include
A) cool, clammy skin
B) bradycardia
C) hypertension
D) constricted pupils.
Question # 193 (Multiple Choice) You arrive at the scene of a bar fight. A 25-year-old man has been
stabbed and is bleeding severely. The assailant is still on the scene and is holding a knife. What should you
do first?
A) Shoot the assailant
B) Stay behind your partner
C) Order the assailant to go away
D) Run to the rig
Question # 194 (Multiple Choice) A 34-year-old female patient has suffered a partial thickness burn to her
forearm. Your treatment should include:
A) applying a burn ointment
B) applying ice
C) puncturing blisters
D) applying a dressing
Question # 195 (Multiple Choice) Regarding abruptio placenta
A) Blood loss is confined within the amniotic sack
B) Internal bleeding is generally minimal.
C) Blood loss may be concealed between the uterine wall and Blood loss may be concealed
between the uterine wall and the placenta
D) There is always excessive external vagina bleeding
Question # 196 (Multiple Choice) Your patient has been crushed between a loading dock and a forklift. He
presents with severe dyspnea, distended neck veins, absent breath sounds on the right side and diminished
breath sounds on the left. You are able to palpate slight tracheal deviation toward the left. You should
initially initial management of this patient includes immediate:
A) perform a pericardiocentesis
B) perform a needle decompression of the right side
C) institute rapid transport
D) perform transtracheal jet insufflation
Question # 197 (Multiple Choice) Your homeless patient presents with bright red rectal bleeding,
tachycardia and hypotension. This is most likely caused by
A) bleeding ulcers
B) esophageal varicies
C) lower GI bleed
D) upper GI bleed
Question # 198 (Multiple Choice) Your patient has a third trimester bleed. You should
A) perform the internal vaginal exam wearing sterile gloves
B) pack the vagina cavity with gauze
C) transport the patient on her left side
D) perform a c-section at the scene
Question # 199 (Multiple Choice) Which of the following forms of hepatitis is generally NOT chronic?
A) Hepatitis A.
B) Hepatitis B
C) Hepatitis C
D) Hepatitis D
Question # 200 (Multiple Choice) Pale, sweaty skin, a rapid thready pulse and a delayed capillary refill time
indicate:
A) ICP
B) hypoperfusion
C) a nervous intern
D) over exertion
Question # 101 (Multiple Choice) You are called for an assault victim. You arrive to find a man
approximately 20 years of age lying on his side in an alley. Examination reveals the patient has blood
oozing from the right side of his nose. The patient's right cheek appears flattened and there is discoloration
around his eyes. The patient states his cheek and nose feel numb. You suspect:
Answer: (D) zygomatic fracture
Question # 102 (Multiple Choice) You are on the scene of a car-pedestrian crash. A 63-year-old woman
was crossing the street when she was struck by a car traveling approximately 45 miles per hour. The patient
was thrown approximately 20 feet. The decision to "load-and-go" or continue stabilization at the scene will
be primarily based on:
Answer: (D) mechanism of injury & physical findings.
Question # 103 (Multiple Choice) Select the correct statement regarding an epidural hematoma.
Answer: (C) In an epidural hematoma, bleeding occurs between the cranium and the dura
Question # 104 (Multiple Choice) A "Halo" test is a check for
Answer: (B) CSF leaks
Question # 105 (Multiple Choice) The METTAG system is one method of triaging patients. When using
this system, a black tag identifies a patient who is:
Answer: (A) dead or unexpected survival.
Question # 106 (Multiple Choice) Responsibilities of the incident commander include:
Answer: (D) rapidly assessing the situation, requesting additional resources as necessary, and developing a
plan of action for management of the incident.
Question # 107 (Multiple Choice) The START method of triage uses a 60-second assessment that
evaluates:
Answer: (C) ventilation, perfusion, mental status
Question # 108 (Multiple Choice) The METTAG system is one method of triaging patients. When using
this system, a green tag identifies a patient who is:
Answer: (B) non-life or limb-threatening.
Question # 109 (Multiple Choice) The START method of triage classifies victims as:
Answer: (A) immediate, delayed, nonsalvageable/dead
Question # 110 (Multiple Choice) The sector responsible for directing emergency vehicles at the scene of a
major incident is the:
Answer: (A) staging sector
Question # 111 (Multiple Choice) A 27-year-old man has suffered a blow to the right side of the head. His
injury may cause compression of the oculomotor cranial nerve resulting in:
Answer: (A) dilation of the ipsilateral pupil.
Question # 112 (Multiple Choice) You are called for an unconscious man. On arrival you find an
unresponsive male, approximately 20 years of age, lying supine in a parking lot. Bystanders state he was the
victim of a gang fight and has been struck in the head and shoulders with a baseball bat. Examination
reveals contusions to the left temporal area and over the shoulders. There is a small amount of bleeding
from the left nostril. The patient's blood pressure is 118/74, pulse 90 and regular, respiratory rate 12/min.
Which of the following represents the MOST IMPORTANT intervention you can perform for this patient to
help minimize cerebral edema?
Answer: (A) Hyperventilate the patient.
Question # 113 (Multiple Choice) The Glasgow Coma Scale evaluates:
Answer: (B) eye opening, motor response, verbal response.
Question # 114 (Multiple Choice) A patient who opens his eyes in response to pain, makes no verbal
response, but withdraws from pain has a Glasgow Coma Score of:
Answer: (C) 7
Question # 115 (Multiple Choice) Cushing's triad is associated with
Answer: (D) increased intracranial pressure
Question # 116 (Multiple Choice) Which of the following signs are associated with Cushing's triad?
Answer: (B) increasing blood pressure, decreasing pulse rate, change in respiratory pattern
Question # 117 (Multiple Choice) Select the most appropriate statement that should be used to describe a
patient's level of consciousness
Answer: (D) The patient withdraws from a painful stimulus
Question # 118 (Multiple Choice) All of the following patients are victims involved in a motor vehicle
crash. Which patient should be rapidly extricated?
Answer: (C) 30-year-old male driver complaining of difficulty breathing. Unable to palpate radial pulse,
carotid weak, rate 120. Respiratory rate 44 and labored
Question # 119 (Multiple Choice) The MOST important sign in the evaluation of a head-injured patient is:
Answer: (D) level of consciousness
Question # 120 (Multiple Choice) LORAZAPAM (ATIVAN) is classified as an
Answer: (A) benzodiazepine
Question # 121 (Multiple Choice) The patient in the "I Feel So Ejected" case was given ATIVAN at the
hospital for
Answer: (C) reducing seizure activity
Question # 122 (Multiple Choice) The correct adult dosage of ATIVAN is
Answer: (A) 0.5 - 2.0 mg IV
Question # 123 (Multiple Choice) The Mannitol dose for an adult is
Answer: (A) 1.5 -2.0 g/kg IV
Question # 124 (Multiple Choice) MANNITOL (OSMOTROL) is an osmotic diuretic that inhibits sodium
and water absorption in the kidneys. It would most likely be used for
Answer: (B) acute cerebral edema
Question # 125 (Multiple Choice) Select the correct statement regarding an epidural hematoma.
Answer: (C) In an epidural hematoma, bleeding occurs between the cranium and the dura.
Question # 126 (Multiple Choice) Signs and symptoms of a basilar skull fracture may include: 1.
Cerebrospinal fluid draining from the nose. 2. Ecchymosis in the mastoid region. 3. Periorbital ecchymosis.
Answer: (D) d. 1, 2, 3.
Question # 127 (Multiple Choice) Nitroglycerin may not be given if the patient
Answer: (B) has a systolic blood pressure less than 90
Question # 128 (Multiple Choice) OPQRST are letters used to help you remember which questions to ask
patients during your assessment. What does the "P" stand for?
Answer: (D) provocation
Question # 129 (Multiple Choice) A 45-year-old patient complains of breathing difficulty and and chest
pain. He will not tolerate a non-rebreather mask. You should
Answer: (A) use a nasal cannula
Question # 130 (Multiple Choice) Your 40-year-old male patient complains of mild chest discomfort. As an
EMT-B you should:
Answer: (C) treat as if the patient has cardiac compromise.
Question # 131 (Multiple Choice) Cardiac compromise commonly includes all of the following signs or
symptoms EXCEPT
Answer: (B) warm, dry skin.
Question # 132 (Multiple Choice) Your adult patient with chest pain has a blood pressure of 92/54, a pulse
of 112 and has a history of angina. You should
Answer: (B) administer the patient's prescribed nitroglycerin
Question # 133 (Multiple Choice) Which of the following is a shockable rythm?
Answer: (B) ventricular fibrillation
Question # 134 (Multiple Choice) Which of the following cardiac arrest patients should you defibrillate?
Answer: (B) a 14-year-old who weighs 110 lbs
Question # 135 (Multiple Answer) All of the following are effects of nitroglycerin (multi answer)
(A) it dilates the blood vessels
(C) it reduces the heart's work load
Question # 136 (Multiple Answer) Which of the following are contraindications for the use of
nitroglycerin? (multi answer)
(C) The patient has also has a head injury
(D) The patient took Viagra 3 hours ago
Question # 137 (Multiple Choice) The cardiac conduction system disturbance that most commonly results
in cardiac arrest is
Answer: (B) ventricular fibrillation
Question # 138 (Multiple Choice) The EMT-B requests prehospital advanced cardiac life support for the
care of the cardiac arrest patient because
Answer: (A) early ACLS intervention provides for higher survival rates
Question # 139 (Multiple Choice) Nitroglycerin is a medication that is administered:
Answer: (C) sublingually.
Question # 140 (Multiple Choice) Why is nitroglycerin effective for patients with recurrent chest pain?
Answer: (C) It dilates coronary blood vessels
Question # 141 (Multiple Choice) Your 55-year-old female patient has chest pain and a blood pressure of
110/80. You give nitroglycerin. You recheck her blood pressure and are unable to obtain a reading. Her
response to the nitroglycerin is called:
Answer: (A) a side effect
Question # 142 (Multiple Choice) Your 55-year-old female patient has chest pain and a blood pressure of
102/70. You assist with one tablet of nitroglycerin. You recheck her blood pressure and it is 80/50 and
become semi- consciousness. A an EMT B, you should
Answer: (D) place her in the Trendelenburg position
Question # 143 (Multiple Choice) Study the picture, and then identify: Structure number 1
Answer: (B) Superior vena cava
Question # 144 (Multiple Choice) Study the picture, and then identify: Structure number 5
Answer: (D) Left ventricle
Question # 145 (Multiple Choice) Using the picture, which number indicates- The first chamber where
blood returns to the heart
Answer: (D) none of the above
Question # 146 (Multiple Choice) If there are no contraindications, ___ mg of ASA should be given to the
chest pain patient.
Answer: (B) 324
Question # 147 (Multiple Choice) The ST segment in lead II and II is
Answer: (B) elevated
Question # 148 (Multiple Choice) A lateral left ventricular myocardial infarction is due to the occlusion of:
Answer: (E) Left circumflex artery
Question # 149 (Multiple Choice) Whichbest refers to the filling and emptying of the heart:
Answer: (B) Starlings Law
Question # 150 (Multiple Choice) The formation of a thrombus
Answer: (D) can lead to an embolism obstructing blood flow in the artery
Question # 151 (Multiple Answer) The left coronary artery supplies (multi answer)
(A) LEFT VENTRICLE
(B) ANTERIOR 2/3 OF THE INTERVENTRICULAR SEPTUM
(C) VARIABLE PORTIONS OF THE RIGHT VENTRICLE
Question # 152 (Multiple Answer) The right coronary artery supplies (multi answer)
(B) SINOATRIAL NODE
(C) POSTERIOR 1/3 OF THE INTERVENTRICULAR SEPTUM
Question # 153 (Multiple Choice) A myocardial infarction occurs when
Answer: (C) there is a blockage in a coronary artery
Question # 154 (Multiple Choice) All of the following are complications of myocardial infarction except
Answer: (D) Dissecting aortic aneurysm
Quuestion # 155 (Multiple Choice) The reciprocal ST that matches the previous answer is seen in leads
Answer: (B) I, aVL and V2
Question # 156 (Multiple Choice) Deep and slurred S waves can be seen in leads
Answer: (A) I, aVL, V5 and V6
Question # 157 (Multiple Choice) Poor R wave progression in the precordial leads along with ST elevation
in V1 could suggest
Answer: (B) the presence of an anterior infarction
Question # 158 (Multiple Choice) Patients with poor cardiac output post nitrogylcerin administration may
need to be treated with an inotropic drug, such as
Answer: (C) dobutamine
Question # 159 (Multiple Choice) Patients with poor cardiac output post nitrogylcerin administration may
need to be treated for symptomatic bradycardia with
Answer: (B) atropine
Question # 160 (Multiple Choice) BONUS QUESTION: Thyroid hyperfunction leads to:
Answer: (B) Marked tachycardia
Question # 161 (Multiple Choice) 1. The currently recognized cause for cat scratch disease is:
Answer: (B) rickettsia
Question # 162 (True/False) 2. True or false, Cat Scratch disease is more common patients under the age of
21
Answer: True
Question # 163 (True/False) 3. Lymphadenopathy in the adult is frequently a sign of metastatic cancer
Answer: True
Question # 164 (Multiple Choice) Cats
Answer: (E) Toxiplasmosis
Question # 165 (Multiple Choice) Rats
Answer: (C) Plague
Question # 166 (Multiple Choice) Cattle
Answer: (D) Brucellosis
Question # 167 (Multiple Choice) Sheep
Answer: (B) Anthrax
Question # 168 (True/False) 5. Lymphadenopathy in children is most commonly associated with infection
Answer: True
Question # 169 (True/False) 6. Declawing of cats is a proper prophylaxis for CSD
Answer: False
Question # 170 (True/False) 7. Humans may acquire CSD directly via bites from infected fleas.
Answer: True
Question # 171 (Multiple Choice) 8. Using the pneumonic AEIOU TIPS as a guide for evaluation of coma,
what does the letter A stand for?
Answer: (A) Acidosis
Question # 172 (Multiple Choice) 9. Causes of seizure GENERALLY include all of the following,
EXCEPT:
Answer: (D) pre-eclampsia
Question # 173 (True/False) 10. Cat bites are more commonly the cause of CSD than scratches
Answer: False
Question # 174 (Multiple Choice) A 54-year-old male patient has been thrown through a window. Glass has
caused an open neck wound. You should dress this wound with:
Answer: (A) an occlusive dressing
Question # 175 (Multiple Choice) Your patient has been exposed to an unknown chemical which has
soaked his clothing. You should flush the patient with
Answer: (A) water
Question # 176 (Multiple Choice) Shock absorption and insulation are the major functions of which layer of
the skin?
Answer: (B) subcutaneous layer
Question # 177 (Multiple Choice) The major problem caused by electrical shock is often not the burn itself.
The most serious problem to consider is
Answer: (C) cardiac arrest
Question # 178 (Multiple Choice) A 23-year-old male patient has been involved in a knife fight and has an
open abdominal injury with protruding bowel. Your treatment of the injury should include
Answer: (C) covering the exposed bowel with a sterile dressing moistened with saline, then an occlusive
dressing
Question # 179 (Multiple Choice) Because it can become a constricting band,______ should NOT use to
hold dressings in place.
Answer: (B) an elastic bandage
Question # 180 (Multiple Choice) A 4-year-old male patient was running in the school hall. You find a
pencil impaled in his upper arm. Your should
Answer: (D) stabilize the pencil with a bulky dressing
Question # 181 (Multiple Choice) A 32-year-old female patient has amputated her index finger in a table
saw accident. You have bandaged her hand. The best way to transport the amputated finger is by
Answer: (A) wrapping the finger in gauze, placing it in a plastic bag, and keeping it cool
Question # 182 (Multiple Choice) A patient has had his lower legs burned. The area is charred black but
your patient states he has little pain. This burn would be classified as
Answer: (D) full thickness
Question # 183 (Multiple Choice) Acceptable care for a superficial burn may include all of the following
EXCEPT
Answer: (A) apply a burn ointment
Question # 184 (Multiple Choice) When force is transmitted to the body's internal structures causing
internal organs to rupture or bleed internally, or small bone fractures to occur is called a(n):
Answer: (A) crush injury
Question # 185 (Multiple Choice) A bruise may be an indication of internal injuries and internal bleeding. It
is important for you to monitor your patient for any signs and symptoms of
Answer: (B) shock
Question # 186 (Multiple Choice) Formation of blood clots would be this mans job :-)
Answer: (B) Plumber Platelet
Question # 187 (Multiple Choice) If a patient is in shock, you would expect the skin to be
Answer: (A) cool and clammy
Question # 188 (Multiple Choice) What does the "D" in "DCAP-BTLS" stand for?
Answer: (D) deformities
Question # 189 (Multiple Choice) A teen male patient has a gunshot wound to the chest. Your detailed
physical exam on this patient
Answer: (C) may never begin due to the seriousness of the chest injury
Question # 190 (Multiple Choice) External bleeding that is a steady flow of dark red blood is from a(n):
Answer: (B) vein
Question # 191 (Multiple Choice) A late sign of internal bleeding is:
Answer: (C) signs of shock
Question # 192 (Multiple Choice) Early signs and symptoms of hypoperfusion include
Answer: (A) cool, clammy skin
Question # 193 (Multiple Choice) You arrive at the scene of a bar fight. A 25-year-old man has been
stabbed and is bleeding severely. The assailant is still on the scene and is holding a knife. What should you
do first?
Answer: (D) Run to the rig
Question # 194 (Multiple Choice) A 34-year-old female patient has suffered a partial thickness burn to her
forearm. Your treatment should include:
Answer: (D) applying a dressing
Question # 195 (Multiple Choice) Regarding abruptio placenta
Answer: (C) Blood loss may be concealed between the uterine wall and Blood loss may be concealed
between the uterine wall and the placenta
Question # 196 (Multiple Choice) Your patient has been crushed between a loading dock and a forklift. He
presents with severe dyspnea, distended neck veins, absent breath sounds on the right side and diminished
breath sounds on the left. You are able to palpate slight tracheal deviation toward the left. You should
initially initial management of this patient includes immediate:
Answer: (B) perform a needle decompression of the right side
Question # 197 (Multiple Choice) Your homeless patient presents with bright red rectal bleeding,
tachycardia and hypotension. This is most likely caused by
Answer: (C) lower GI bleed
Question # 198 (Multiple Choice) Your patient has a third trimester bleed. You should
Answer: (C) transport the patient on her left side
Question # 199 (Multiple Choice) Which of the following forms of hepatitis is generally NOT chronic?
Answer: (A) Hepatitis A.
Question # 200 (Multiple Choice) Pale, sweaty skin, a rapid thready pulse and a delayed capillary refill time
indicate:
Answer: (B) hypoperfusion

More Related Content

What's hot

What's hot (20)

Trauma
TraumaTrauma
Trauma
 
Preoperative managment
Preoperative managment Preoperative managment
Preoperative managment
 
Principles of preoperative and operative surgery
Principles of preoperative and operative surgeryPrinciples of preoperative and operative surgery
Principles of preoperative and operative surgery
 
Cvp line
Cvp lineCvp line
Cvp line
 
Neurogenic shock
Neurogenic shockNeurogenic shock
Neurogenic shock
 
Endotracheal intubation
Endotracheal intubationEndotracheal intubation
Endotracheal intubation
 
Post op care
Post op carePost op care
Post op care
 
Tracheostomy
TracheostomyTracheostomy
Tracheostomy
 
Decompressive craniectomy in Traumatic Brain Injury
Decompressive craniectomy in Traumatic Brain InjuryDecompressive craniectomy in Traumatic Brain Injury
Decompressive craniectomy in Traumatic Brain Injury
 
ATLS (Advance Trauma Life Support)
ATLS (Advance Trauma Life Support)ATLS (Advance Trauma Life Support)
ATLS (Advance Trauma Life Support)
 
Disaster surgery- triage
Disaster surgery- triageDisaster surgery- triage
Disaster surgery- triage
 
Secondary survey in trauma
Secondary survey in traumaSecondary survey in trauma
Secondary survey in trauma
 
Surgery mcq
Surgery mcqSurgery mcq
Surgery mcq
 
Damage control surgery
Damage  control  surgeryDamage  control  surgery
Damage control surgery
 
Primary survey in Trauma
Primary survey in TraumaPrimary survey in Trauma
Primary survey in Trauma
 
Perioperative Management
Perioperative ManagementPerioperative Management
Perioperative Management
 
Intercostal drainage
Intercostal drainageIntercostal drainage
Intercostal drainage
 
Preoperative preparation
Preoperative preparationPreoperative preparation
Preoperative preparation
 
Intra operative care.pptx
Intra operative care.pptxIntra operative care.pptx
Intra operative care.pptx
 
Ventilator associated pneumonia VAP
Ventilator associated pneumonia VAPVentilator associated pneumonia VAP
Ventilator associated pneumonia VAP
 

Similar to 100 question test variety 2

Surgery 209 krok 2
Surgery 209 krok 2Surgery 209 krok 2
Surgery 209 krok 2Raj Twix
 
Therapeutic 176 additional krok 2
Therapeutic 176 additional krok 2Therapeutic 176 additional krok 2
Therapeutic 176 additional krok 2Raj Twix
 
Coronary artery disease
Coronary artery diseaseCoronary artery disease
Coronary artery diseasehuma2012
 
Review course 2013 answer key.anand.tiwari
Review course 2013 answer key.anand.tiwariReview course 2013 answer key.anand.tiwari
Review course 2013 answer key.anand.tiwariAnand Tiwari
 
Medical surgical nursing quiz
Medical surgical nursing quizMedical surgical nursing quiz
Medical surgical nursing quizNursing Path
 
Book 2011 krok 2
Book 2011 krok 2Book 2011 krok 2
Book 2011 krok 2Raj Twix
 
Surgey addi krok 2
Surgey addi krok 2Surgey addi krok 2
Surgey addi krok 2Raj Twix
 
Oncologcial Emergencies by Prof Ahmed Badheeb 2014 part 1
Oncologcial  Emergencies by Prof Ahmed Badheeb 2014 part 1Oncologcial  Emergencies by Prof Ahmed Badheeb 2014 part 1
Oncologcial Emergencies by Prof Ahmed Badheeb 2014 part 1Prof. Ahmed Mohamed Badheeb
 
Medical surgical nursing quiz
Medical surgical nursing quizMedical surgical nursing quiz
Medical surgical nursing quizNursing Path
 
Drs. Escobar, Pikus, and Blackwell’s CMC X-Ray Mastery Project: 43rd Case Series
Drs. Escobar, Pikus, and Blackwell’s CMC X-Ray Mastery Project: 43rd Case SeriesDrs. Escobar, Pikus, and Blackwell’s CMC X-Ray Mastery Project: 43rd Case Series
Drs. Escobar, Pikus, and Blackwell’s CMC X-Ray Mastery Project: 43rd Case SeriesSean M. Fox
 
Emergency nursing questionnaires
Emergency nursing questionnairesEmergency nursing questionnaires
Emergency nursing questionnairesJoan Delgado
 
Assessment and management of trauma
Assessment and management of traumaAssessment and management of trauma
Assessment and management of traumaJoginder Singh
 
Therapy 2013 krok 2
Therapy 2013 krok 2Therapy 2013 krok 2
Therapy 2013 krok 2Raj Twix
 

Similar to 100 question test variety 2 (20)

Clinical cases (1) june 30, 13
Clinical cases  (1) june 30, 13Clinical cases  (1) june 30, 13
Clinical cases (1) june 30, 13
 
Surgery 209 krok 2
Surgery 209 krok 2Surgery 209 krok 2
Surgery 209 krok 2
 
Therapeutic 176 additional krok 2
Therapeutic 176 additional krok 2Therapeutic 176 additional krok 2
Therapeutic 176 additional krok 2
 
Coronary artery disease
Coronary artery diseaseCoronary artery disease
Coronary artery disease
 
Review course 2013 answer key.anand.tiwari
Review course 2013 answer key.anand.tiwariReview course 2013 answer key.anand.tiwari
Review course 2013 answer key.anand.tiwari
 
Medical surgical nursing quiz
Medical surgical nursing quizMedical surgical nursing quiz
Medical surgical nursing quiz
 
Quiz for BDS, OMFS
Quiz for BDS, OMFSQuiz for BDS, OMFS
Quiz for BDS, OMFS
 
Book 2011 krok 2
Book 2011 krok 2Book 2011 krok 2
Book 2011 krok 2
 
Surgey addi krok 2
Surgey addi krok 2Surgey addi krok 2
Surgey addi krok 2
 
Oncologcial Emergencies by Prof Ahmed Badheeb 2014 part 1
Oncologcial  Emergencies by Prof Ahmed Badheeb 2014 part 1Oncologcial  Emergencies by Prof Ahmed Badheeb 2014 part 1
Oncologcial Emergencies by Prof Ahmed Badheeb 2014 part 1
 
Medical surgical nursing quiz
Medical surgical nursing quizMedical surgical nursing quiz
Medical surgical nursing quiz
 
Clinical cases (1) july 7, 13
Clinical cases  (1) july 7, 13Clinical cases  (1) july 7, 13
Clinical cases (1) july 7, 13
 
NIMSET 2013 Questions
NIMSET 2013 QuestionsNIMSET 2013 Questions
NIMSET 2013 Questions
 
Medical Surgical Nursing
Medical Surgical NursingMedical Surgical Nursing
Medical Surgical Nursing
 
Drs. Escobar, Pikus, and Blackwell’s CMC X-Ray Mastery Project: 43rd Case Series
Drs. Escobar, Pikus, and Blackwell’s CMC X-Ray Mastery Project: 43rd Case SeriesDrs. Escobar, Pikus, and Blackwell’s CMC X-Ray Mastery Project: 43rd Case Series
Drs. Escobar, Pikus, and Blackwell’s CMC X-Ray Mastery Project: 43rd Case Series
 
Segundo simulador primera parte
Segundo simulador primera parteSegundo simulador primera parte
Segundo simulador primera parte
 
Medicine BCQs (Nervous system)
Medicine BCQs (Nervous system)Medicine BCQs (Nervous system)
Medicine BCQs (Nervous system)
 
Emergency nursing questionnaires
Emergency nursing questionnairesEmergency nursing questionnaires
Emergency nursing questionnaires
 
Assessment and management of trauma
Assessment and management of traumaAssessment and management of trauma
Assessment and management of trauma
 
Therapy 2013 krok 2
Therapy 2013 krok 2Therapy 2013 krok 2
Therapy 2013 krok 2
 

More from Al-YAQIN DIAGNOSTIC ULTRASONIC CLINIC BAGHDAD

More from Al-YAQIN DIAGNOSTIC ULTRASONIC CLINIC BAGHDAD (20)

Pediatric Emergency Medicine 2019.pdf
Pediatric Emergency Medicine 2019.pdfPediatric Emergency Medicine 2019.pdf
Pediatric Emergency Medicine 2019.pdf
 
الأمراض الجلدية للأطفال.pdf
الأمراض الجلدية للأطفال.pdfالأمراض الجلدية للأطفال.pdf
الأمراض الجلدية للأطفال.pdf
 
Clinical-Skills-for-OSCEs.pdf
Clinical-Skills-for-OSCEs.pdfClinical-Skills-for-OSCEs.pdf
Clinical-Skills-for-OSCEs.pdf
 
Hematology learning guide 1
Hematology learning guide 1 Hematology learning guide 1
Hematology learning guide 1
 
Introduction to heamatology
Introduction to heamatologyIntroduction to heamatology
Introduction to heamatology
 
Chronic leukemia
  Chronic leukemia    Chronic leukemia
Chronic leukemia
 
Drug intoxication
Drug intoxicationDrug intoxication
Drug intoxication
 
stomach surgery for medical students
stomach surgery for medical students stomach surgery for medical students
stomach surgery for medical students
 
Radiographic Evaluation of Interstitial Lung Disease Laura E. Heyneman, MD Du...
Radiographic Evaluation of Interstitial Lung Disease Laura E. Heyneman, MD Du...Radiographic Evaluation of Interstitial Lung Disease Laura E. Heyneman, MD Du...
Radiographic Evaluation of Interstitial Lung Disease Laura E. Heyneman, MD Du...
 
Typhoid fever
Typhoid feverTyphoid fever
Typhoid fever
 
Human Herpesviruses INFECTIOUS MONONUCLEOSIS 2016
Human Herpesviruses  INFECTIOUS MONONUCLEOSIS 2016Human Herpesviruses  INFECTIOUS MONONUCLEOSIS 2016
Human Herpesviruses INFECTIOUS MONONUCLEOSIS 2016
 
Chickenpox
ChickenpoxChickenpox
Chickenpox
 
Vzv infections
Vzv infectionsVzv infections
Vzv infections
 
Plague kaki 20161130
Plague kaki 20161130Plague kaki 20161130
Plague kaki 20161130
 
Rabies - Dr Akaki 20161129
Rabies - Dr Akaki 20161129Rabies - Dr Akaki 20161129
Rabies - Dr Akaki 20161129
 
Surgical disorder of spleen --basheer oudah
Surgical disorder of spleen   --basheer oudahSurgical disorder of spleen   --basheer oudah
Surgical disorder of spleen --basheer oudah
 
Systematic error bias
Systematic error  biasSystematic error  bias
Systematic error bias
 
Walraven article
Walraven articleWalraven article
Walraven article
 
Viraltropism
ViraltropismViraltropism
Viraltropism
 
Plague kaki 2016 11 30
Plague kaki 2016 11 30Plague kaki 2016 11 30
Plague kaki 2016 11 30
 

Recently uploaded

Hybridoma Technology ( Production , Purification , and Application )
Hybridoma Technology  ( Production , Purification , and Application  ) Hybridoma Technology  ( Production , Purification , and Application  )
Hybridoma Technology ( Production , Purification , and Application ) Sakshi Ghasle
 
Incoming and Outgoing Shipments in 1 STEP Using Odoo 17
Incoming and Outgoing Shipments in 1 STEP Using Odoo 17Incoming and Outgoing Shipments in 1 STEP Using Odoo 17
Incoming and Outgoing Shipments in 1 STEP Using Odoo 17Celine George
 
The Most Excellent Way | 1 Corinthians 13
The Most Excellent Way | 1 Corinthians 13The Most Excellent Way | 1 Corinthians 13
The Most Excellent Way | 1 Corinthians 13Steve Thomason
 
Kisan Call Centre - To harness potential of ICT in Agriculture by answer farm...
Kisan Call Centre - To harness potential of ICT in Agriculture by answer farm...Kisan Call Centre - To harness potential of ICT in Agriculture by answer farm...
Kisan Call Centre - To harness potential of ICT in Agriculture by answer farm...Krashi Coaching
 
Presentation by Andreas Schleicher Tackling the School Absenteeism Crisis 30 ...
Presentation by Andreas Schleicher Tackling the School Absenteeism Crisis 30 ...Presentation by Andreas Schleicher Tackling the School Absenteeism Crisis 30 ...
Presentation by Andreas Schleicher Tackling the School Absenteeism Crisis 30 ...EduSkills OECD
 
URLs and Routing in the Odoo 17 Website App
URLs and Routing in the Odoo 17 Website AppURLs and Routing in the Odoo 17 Website App
URLs and Routing in the Odoo 17 Website AppCeline George
 
Introduction to ArtificiaI Intelligence in Higher Education
Introduction to ArtificiaI Intelligence in Higher EducationIntroduction to ArtificiaI Intelligence in Higher Education
Introduction to ArtificiaI Intelligence in Higher Educationpboyjonauth
 
Solving Puzzles Benefits Everyone (English).pptx
Solving Puzzles Benefits Everyone (English).pptxSolving Puzzles Benefits Everyone (English).pptx
Solving Puzzles Benefits Everyone (English).pptxOH TEIK BIN
 
A Critique of the Proposed National Education Policy Reform
A Critique of the Proposed National Education Policy ReformA Critique of the Proposed National Education Policy Reform
A Critique of the Proposed National Education Policy ReformChameera Dedduwage
 
Q4-W6-Restating Informational Text Grade 3
Q4-W6-Restating Informational Text Grade 3Q4-W6-Restating Informational Text Grade 3
Q4-W6-Restating Informational Text Grade 3JemimahLaneBuaron
 
Mastering the Unannounced Regulatory Inspection
Mastering the Unannounced Regulatory InspectionMastering the Unannounced Regulatory Inspection
Mastering the Unannounced Regulatory InspectionSafetyChain Software
 
Contemporary philippine arts from the regions_PPT_Module_12 [Autosaved] (1).pptx
Contemporary philippine arts from the regions_PPT_Module_12 [Autosaved] (1).pptxContemporary philippine arts from the regions_PPT_Module_12 [Autosaved] (1).pptx
Contemporary philippine arts from the regions_PPT_Module_12 [Autosaved] (1).pptxRoyAbrique
 
APM Welcome, APM North West Network Conference, Synergies Across Sectors
APM Welcome, APM North West Network Conference, Synergies Across SectorsAPM Welcome, APM North West Network Conference, Synergies Across Sectors
APM Welcome, APM North West Network Conference, Synergies Across SectorsAssociation for Project Management
 
Measures of Central Tendency: Mean, Median and Mode
Measures of Central Tendency: Mean, Median and ModeMeasures of Central Tendency: Mean, Median and Mode
Measures of Central Tendency: Mean, Median and ModeThiyagu K
 
MENTAL STATUS EXAMINATION format.docx
MENTAL     STATUS EXAMINATION format.docxMENTAL     STATUS EXAMINATION format.docx
MENTAL STATUS EXAMINATION format.docxPoojaSen20
 
Organic Name Reactions for the students and aspirants of Chemistry12th.pptx
Organic Name Reactions  for the students and aspirants of Chemistry12th.pptxOrganic Name Reactions  for the students and aspirants of Chemistry12th.pptx
Organic Name Reactions for the students and aspirants of Chemistry12th.pptxVS Mahajan Coaching Centre
 
Alper Gobel In Media Res Media Component
Alper Gobel In Media Res Media ComponentAlper Gobel In Media Res Media Component
Alper Gobel In Media Res Media ComponentInMediaRes1
 
Crayon Activity Handout For the Crayon A
Crayon Activity Handout For the Crayon ACrayon Activity Handout For the Crayon A
Crayon Activity Handout For the Crayon AUnboundStockton
 

Recently uploaded (20)

Hybridoma Technology ( Production , Purification , and Application )
Hybridoma Technology  ( Production , Purification , and Application  ) Hybridoma Technology  ( Production , Purification , and Application  )
Hybridoma Technology ( Production , Purification , and Application )
 
Incoming and Outgoing Shipments in 1 STEP Using Odoo 17
Incoming and Outgoing Shipments in 1 STEP Using Odoo 17Incoming and Outgoing Shipments in 1 STEP Using Odoo 17
Incoming and Outgoing Shipments in 1 STEP Using Odoo 17
 
The Most Excellent Way | 1 Corinthians 13
The Most Excellent Way | 1 Corinthians 13The Most Excellent Way | 1 Corinthians 13
The Most Excellent Way | 1 Corinthians 13
 
Kisan Call Centre - To harness potential of ICT in Agriculture by answer farm...
Kisan Call Centre - To harness potential of ICT in Agriculture by answer farm...Kisan Call Centre - To harness potential of ICT in Agriculture by answer farm...
Kisan Call Centre - To harness potential of ICT in Agriculture by answer farm...
 
Presentation by Andreas Schleicher Tackling the School Absenteeism Crisis 30 ...
Presentation by Andreas Schleicher Tackling the School Absenteeism Crisis 30 ...Presentation by Andreas Schleicher Tackling the School Absenteeism Crisis 30 ...
Presentation by Andreas Schleicher Tackling the School Absenteeism Crisis 30 ...
 
URLs and Routing in the Odoo 17 Website App
URLs and Routing in the Odoo 17 Website AppURLs and Routing in the Odoo 17 Website App
URLs and Routing in the Odoo 17 Website App
 
Introduction to ArtificiaI Intelligence in Higher Education
Introduction to ArtificiaI Intelligence in Higher EducationIntroduction to ArtificiaI Intelligence in Higher Education
Introduction to ArtificiaI Intelligence in Higher Education
 
Model Call Girl in Bikash Puri Delhi reach out to us at 🔝9953056974🔝
Model Call Girl in Bikash Puri  Delhi reach out to us at 🔝9953056974🔝Model Call Girl in Bikash Puri  Delhi reach out to us at 🔝9953056974🔝
Model Call Girl in Bikash Puri Delhi reach out to us at 🔝9953056974🔝
 
Solving Puzzles Benefits Everyone (English).pptx
Solving Puzzles Benefits Everyone (English).pptxSolving Puzzles Benefits Everyone (English).pptx
Solving Puzzles Benefits Everyone (English).pptx
 
A Critique of the Proposed National Education Policy Reform
A Critique of the Proposed National Education Policy ReformA Critique of the Proposed National Education Policy Reform
A Critique of the Proposed National Education Policy Reform
 
Staff of Color (SOC) Retention Efforts DDSD
Staff of Color (SOC) Retention Efforts DDSDStaff of Color (SOC) Retention Efforts DDSD
Staff of Color (SOC) Retention Efforts DDSD
 
Q4-W6-Restating Informational Text Grade 3
Q4-W6-Restating Informational Text Grade 3Q4-W6-Restating Informational Text Grade 3
Q4-W6-Restating Informational Text Grade 3
 
Mastering the Unannounced Regulatory Inspection
Mastering the Unannounced Regulatory InspectionMastering the Unannounced Regulatory Inspection
Mastering the Unannounced Regulatory Inspection
 
Contemporary philippine arts from the regions_PPT_Module_12 [Autosaved] (1).pptx
Contemporary philippine arts from the regions_PPT_Module_12 [Autosaved] (1).pptxContemporary philippine arts from the regions_PPT_Module_12 [Autosaved] (1).pptx
Contemporary philippine arts from the regions_PPT_Module_12 [Autosaved] (1).pptx
 
APM Welcome, APM North West Network Conference, Synergies Across Sectors
APM Welcome, APM North West Network Conference, Synergies Across SectorsAPM Welcome, APM North West Network Conference, Synergies Across Sectors
APM Welcome, APM North West Network Conference, Synergies Across Sectors
 
Measures of Central Tendency: Mean, Median and Mode
Measures of Central Tendency: Mean, Median and ModeMeasures of Central Tendency: Mean, Median and Mode
Measures of Central Tendency: Mean, Median and Mode
 
MENTAL STATUS EXAMINATION format.docx
MENTAL     STATUS EXAMINATION format.docxMENTAL     STATUS EXAMINATION format.docx
MENTAL STATUS EXAMINATION format.docx
 
Organic Name Reactions for the students and aspirants of Chemistry12th.pptx
Organic Name Reactions  for the students and aspirants of Chemistry12th.pptxOrganic Name Reactions  for the students and aspirants of Chemistry12th.pptx
Organic Name Reactions for the students and aspirants of Chemistry12th.pptx
 
Alper Gobel In Media Res Media Component
Alper Gobel In Media Res Media ComponentAlper Gobel In Media Res Media Component
Alper Gobel In Media Res Media Component
 
Crayon Activity Handout For the Crayon A
Crayon Activity Handout For the Crayon ACrayon Activity Handout For the Crayon A
Crayon Activity Handout For the Crayon A
 

100 question test variety 2

  • 1. Question # 101 (Multiple Choice) You are called for an assault victim. You arrive to find a man approximately 20 years of age lying on his side in an alley. Examination reveals the patient has blood oozing from the right side of his nose. The patient's right cheek appears flattened and there is discoloration around his eyes. The patient states his cheek and nose feel numb. You suspect: A) mandibular fracture B) basilar skull fracture C) parotid fracture D) zygomatic fracture Question # 102 (Multiple Choice) You are on the scene of a car-pedestrian crash. A 63-year-old woman was crossing the street when she was struck by a car traveling approximately 45 miles per hour. The patient was thrown approximately 20 feet. The decision to "load-and-go" or continue stabilization at the scene will be primarily based on: A) local triage protocols B) national trauma guidelines. C) employer policies & physical findings D) mechanism of injury & physical findings. Question # 103 (Multiple Choice) Select the correct statement regarding an epidural hematoma. A) In an epidural hematoma, bleeding occurs between the dura and arachnoid mater. B) An epidural hematoma develops slowly, producing symptoms hours or days after the injury. C) In an epidural hematoma, bleeding occurs between the cranium and the dura D) An epidural hematoma is most often associated with venous bleeding Question # 104 (Multiple Choice) A "Halo" test is a check for A) cervical injury B) CSF leaks C) Priapism D) Brain damage Question # 105 (Multiple Choice) The METTAG system is one method of triaging patients. When using this system, a black tag identifies a patient who is:
  • 2. A) dead or unexpected survival. B) non-life or limb-threatening. C) less critically injured. D) the most critically injured. Question # 106 (Multiple Choice) Responsibilities of the incident commander include: A) working with the communication center and hospitals to obtain medical facility status and treatment capabilities. B) evaluating the resources required for patient treatment and providing suitable "immediate" and "delayed" treatment areas. C) determining the resources necessary to extricate trapped patients and delivering them to the treatment sector. D) rapidly assessing the situation, requesting additional resources as necessary, and developing a plan of action for management of the incident. Question # 107 (Multiple Choice) The START method of triage uses a 60-second assessment that evaluates: A) pulses, motor, sensation B) airway, breathing, circulation C) ventilation, perfusion, mental status D) level of consciousness, tenderness, crepitation Question # 108 (Multiple Choice) The METTAG system is one method of triaging patients. When using this system, a green tag identifies a patient who is: A) dead or unexpected survival. B) non-life or limb-threatening. C) less critically injured. D) the most critically injured. Question # 109 (Multiple Choice) The START method of triage classifies victims as: A) immediate, delayed, nonsalvageable/dead B) delayed, non-life or limb-threatening, dead C) the most critically injured, walking wounded, dead D) walking wounded, nonsalvageable, the most critically injured
  • 3. Question # 110 (Multiple Choice) The sector responsible for directing emergency vehicles at the scene of a major incident is the: A) staging sector B) treatment sector C) extrication sector D) transportation sector Question # 111 (Multiple Choice) A 27-year-old man has suffered a blow to the right side of the head. His injury may cause compression of the oculomotor cranial nerve resulting in: A) dilation of the ipsilateral pupil. B) dilation of the contralateral pupil. C) constriction of the ipsilateral pupil. D) constriction of the contralateral pupil. Question # 112 (Multiple Choice) You are called for an unconscious man. On arrival you find an unresponsive male, approximately 20 years of age, lying supine in a parking lot. Bystanders state he was the victim of a gang fight and has been struck in the head and shoulders with a baseball bat. Examination reveals contusions to the left temporal area and over the shoulders. There is a small amount of bleeding from the left nostril. The patient's blood pressure is 118/74, pulse 90 and regular, respiratory rate 12/min. Which of the following represents the MOST IMPORTANT intervention you can perform for this patient to help minimize cerebral edema? A) Hyperventilate the patient. B) Apply the pneumatic antishock garment. C) Placing the patient in Trendelenburg position. D) Establish two large-bore IVs of normal saline and infuse wide open. Question # 113 (Multiple Choice) The Glasgow Coma Scale evaluates: A) motor response, gag reflex, verbal response. B) eye opening, motor response, verbal response. C) eye opening, pupillary response, motor response. D) verbal response, pupillary response, motor response. Question # 114 (Multiple Choice) A patient who opens his eyes in response to pain, makes no verbal response, but withdraws from pain has a Glasgow Coma Score of:
  • 4. A) 3 B) 5 C) 7 D) 11 Question # 115 (Multiple Choice) Cushing's triad is associated with A) cardiac tamponade B) tension pneumothorax C) massive hemothorax D) increased intracranial pressure Question # 116 (Multiple Choice) Which of the following signs are associated with Cushing's triad? A) decreasing blood pressure, decreasing pulse rate, change in respiratory pattern B) increasing blood pressure, decreasing pulse rate, change in respiratory pattern C) increasing blood pressure, increasing pulse rate, change in respiratory pattern D) decreasing blood pressure, increasing pulse rate, change in respiratory pattern Question # 117 (Multiple Choice) Select the most appropriate statement that should be used to describe a patient's level of consciousness A) The patient is obtunded B) The patient appears drowsy C) The patient is semiconscious D) The patient withdraws from a painful stimulus Question # 118 (Multiple Choice) All of the following patients are victims involved in a motor vehicle crash. Which patient should be rapidly extricated? A) 26-year-old female front seat passenger complaining of a headache. Laceration present on right forehead. BP 138/80, P 92, R 16 B) 19-year-old male back seat passenger complaining of pain in right femur. Swelling and discoloration noted over right thigh. BP 128/74, P 88, R 14 C) 30-year-old male driver complaining of difficulty breathing. Unable to palpate radial pulse, carotid weak, rate 120. Respiratory rate 44 and labored D) 24-year-old female back seat passenger complaining of abdominal pain. Seatbelt markings noted over chest and abdomen. BP 118/82, P 76, R 16
  • 5. Question # 119 (Multiple Choice) The MOST important sign in the evaluation of a head-injured patient is: A) tachycardia B) blood pressure C) tachypnea D) level of consciousness Question # 120 (Multiple Choice) LORAZAPAM (ATIVAN) is classified as an A) benzodiazepine B) vasodilator C) anit-inflammatory agent D) anticholinergic Question # 121 (Multiple Choice) The patient in the "I Feel So Ejected" case was given ATIVAN at the hospital for A) reducing brain swelling B) reducing hypotenison C) reducing seizure activity D) reducing PVC's Question # 122 (Multiple Choice) The correct adult dosage of ATIVAN is A) 0.5 - 2.0 mg IV B) 1-2 g diluted in 10 ml of D5W C) 2-4 g IV D) 1.5 -2.0 g/kg IV Question # 123 (Multiple Choice) The Mannitol dose for an adult is A) 1.5 -2.0 g/kg IV B) 25-50 mg IV C) 0.5 mg/lb to 0.8 mg/lb IM, SQ D) 0.2 ml in 2.5 ml saline Question # 124 (Multiple Choice) MANNITOL (OSMOTROL) is an osmotic diuretic that inhibits sodium and water absorption in the kidneys. It would most likely be used for
  • 6. A) GI bleeds B) acute cerebral edema C) kidney injury D) Loop of Henley dysfunction Question # 125 (Multiple Choice) Select the correct statement regarding an epidural hematoma. A) In an epidural hematoma, bleeding occurs between the dura and arachnoid mater. B) n epidural hematoma develops slowly, producing symptoms hours or days after the injury. C) In an epidural hematoma, bleeding occurs between the cranium and the dura. D) An epidural hematoma is most often associated with venous bleeding. Question # 126 (Multiple Choice) Signs and symptoms of a basilar skull fracture may include: 1. Cerebrospinal fluid draining from the nose. 2. Ecchymosis in the mastoid region. 3. Periorbital ecchymosis. A) a. 1, 2 only. B) b. 1, 3 only. C) c. 2, 3 only. D) d. 1, 2, 3. Question # 127 (Multiple Choice) Nitroglycerin may not be given if the patient A) has used two tablets of nitroglycerin B) has a systolic blood pressure less than 90 C) has a diastolic blood pressure less than 90 D) has taken one tablet with no relief Question # 128 (Multiple Choice) OPQRST are letters used to help you remember which questions to ask patients during your assessment. What does the "P" stand for? A) proximity B) pulse C) pain D) provocation Question # 129 (Multiple Choice) A 45-year-old patient complains of breathing difficulty and and chest pain. He will not tolerate a non-rebreather mask. You should
  • 7. A) use a nasal cannula B) move the mask to a position of comfort C) provide artificial ventilation with a bag-valve mask D) use a pocket mask with supplemental oxygen Question # 130 (Multiple Choice) Your 40-year-old male patient complains of mild chest discomfort. As an EMT-B you should: A) decide whether or not the patient has a heart problem B) decide what type of heart problem it might be C) treat as if the patient has cardiac compromise. D) all of the above Question # 131 (Multiple Choice) Cardiac compromise commonly includes all of the following signs or symptoms EXCEPT A) difficulty breathing. B) warm, dry skin. C) nausea or vomiting. D) epigastric pain Question # 132 (Multiple Choice) Your adult patient with chest pain has a blood pressure of 92/54, a pulse of 112 and has a history of angina. You should A) transport the patient promptly. B) administer the patient's prescribed nitroglycerin C) quickly begin CPR/full resuscitation efforts D) apply an automated external defibrillator and monitor the patient's cardiac rhythm Question # 133 (Multiple Choice) Which of the following is a shockable rythum? A) sinus rhythm B) ventricular fibrillation C) pulseless electrical activity D) third degree block Question # 134 (Multiple Choice) Which of the following cardiac arrest patients should you defibrillate?
  • 8. A) a 12-year-old who weighs less 90 lbs B) a 14-year-old who weighs 110 lbs C) a 12-year-old trauma patient who weighs 120 lbs D) an infant experiencing sudden infant death syndrome Question # 135 (Multiple Answer) All of the following are effects of nitroglycerin (multi answer) A) it dilates the blood vessels B) it allows more blood to remain in the capillaries C) it reduces the heart's work load D) it increases the blood pressure Question # 136 (Multiple Answer) Which of the following are contraindications for the use of nitroglycerin? (multi answer) A) The patient has a history of cardiac problems B) The patient complains of chest pain C) The patient has also has a head injury D) The patient took Viagra 3 hours ago Question # 137 (Multiple Choice) The cardiac conduction system disturbance that most commonly results in cardiac arrest is A) pulseless electrical activity B) ventricular fibrillation C) ventricular tachycardia D) asystole. Question # 138 (Multiple Choice) The EMT-B requests prehospital advanced cardiac life support for the care of the cardiac arrest patient because A) early ACLS intervention provides for higher survival rates B) ACLS personnel must be present when EMT-Bs perform defibrillation. C) only paramedics can transport cardiac arrest patients D) the EMT-B is not adequately trained to manage cardiac arrest Question # 139 (Multiple Choice) Nitroglycerin is a medication that is administered:
  • 9. A) as a slurry B) as an injection C) sublingually. D) by inhalation Question # 140 (Multiple Choice) Why is nitroglycerin effective for patients with recurrent chest pain? A) It enlarges bronchial tubes B) It constricts the blood vessels C) It dilates coronary blood vessels D) All of the above Question # 141 (Multiple Choice) Your 55-year-old female patient has chest pain and a blood pressure of 110/80. You give nitroglycerin. You recheck her blood pressure and are unable to obtain a reading. Her response to the nitroglycerin is called: A) a side effect B) an indication C) a contraindication D) a characteristic Question # 142 (Multiple Choice) Your 55-year-old female patient has chest pain and a blood pressure of 102/70. You assist with one tablet of nitroglycerin. You recheck her blood pressure and it is 80/50 and become semi- consciousness. A an EMT B, you should A) place her in the Fowlers position B) place her in the semi reverse Trendelenburg position C) place her in the semi Fowlers position D) place her in the Trendelenburg position Question # 143 (Multiple Choice) Study the picture, and then identify: Structure number 1 A) Inferior vena cava B) Superior vena cava C) Aorta D) Right ventricle Question # 144 (Multiple Choice) Study the picture, and then identify: Structure number 5
  • 10. A) Right ventricle B) Aorta C) Inferior vena cava D) Left ventricle Question # 145 (Multiple Choice) Using the picture, which number indicates- The first chamber where blood returns to the heart A) 1 B) 2 C) 5 D) none of the above Question # 146 (Multiple Choice) If there are no contraindications, ___ mg of ASA should be given to the chest pain patient. A) 362 B) 324 C) 400 D) 320 Question # 147 (Multiple Choice) The ST segment in lead II and II is A) depressed B) elevated C) normal D) isoelectric Question # 148 (Multiple Choice) A lateral left ventricular myocardial infarction is due to the occlusion of: A) Perforating arteries B) Right coronary artery C) Right marginal branch D) Left anterior descending E) Left circumflex artery Question # 149 (Multiple Choice) Whichbest refers to the filling and emptying of the heart:
  • 11. A) Blood Brain Barrier B) Starlings Law C) Einthoven's Triangle D) Angle of Louis E) Poiseulle's Law Question # 150 (Multiple Choice) The formation of a thrombus A) is the result of rupture of the artery wall B) a thinning of the artery wall C) is the result of myocardial cells becoming ischemic D) can lead to an embolism obstructing blood flow in the artery Question # 151 (Multiple Answer) The left coronary artery supplies (multi answer) A) LEFT VENTRICLE B) ANTERIOR 2/3 OF THE INTERVENTRICULAR SEPTUM C) VARIABLE PORTIONS OF THE RIGHT VENTRICLE D) ATRIOVENTRICULAR NODE Question # 152 (Multiple Answer) The right coronary artery supplies (multi answer) A) POSTEROLATERAL SURFACE OF THE HEART B) SINOATRIAL NODE C) POSTERIOR 1/3 OF THE INTERVENTRICULAR SEPTUM D) LEFT VENTRICLE Question # 153 (Multiple Choice) A myocardial infarction occurs when A) a heart valve does not close completely B) a heart valve does not open fully C) there is a blockage in a coronary artery D) the electrical impulses of the heart become irregular Question # 154 (Multiple Choice) All of the following are complications of myocardial infarction except
  • 12. A) Cor pulmonale B) Cerebral emboli C) Cardiac tamponade D) Dissecting aortic aneurysm Question # 155 (Multiple Choice) The reciprocal ST that matches the previous answer is seen in leads A) aVL and V2 B) I, aVL and V2 C) aVL, V5 and V6 D) aVF, aVL and V2 Question # 156 (Multiple Choice) Deep and slurred S waves can be seen in leads A) I, aVL, V5 and V6 B) V1 to V4 C) I, aVL and V2 Question # 157 (Multiple Choice) Poor R wave progression in the precordial leads along with ST elevation in V1 could suggest A) the presence of a septal infarction B) the presence of an anterior infarction C) the presence of an aortic tear D) the presence of Cor pulmonale Question # 158 (Multiple Choice) Patients with poor cardiac output post nitrogylcerin administration may need to be treated with an inotropic drug, such as A) lidocaine B) atropine C) dobutamine D) mag sulfate Question # 159 (Multiple Choice) Patients with poor cardiac output post nitrogylcerin administration may need to be treated for symptomatic bradycardia with
  • 13. A) lidocaine B) atropine C) dobutamine D) mag sulfate Question # 160 (Multiple Choice) BONUS QUESTION: Thyroid hyperfunction leads to: A) Mitral stenosis B) Marked tachycardia C) Decreased cardiac output D) Right-sided heart failure E) Increased peripheral vascular resistance Question # 161 (Multiple Choice) 1. The currently recognized cause for cat scratch disease is: A) prion B) rickettsia C) fungus D) virus Question # 162 (True/False) 2. True or false, Cat Scratch disease is more common patients under the age of 21 A) True B) False Question # 163 (True/False) 3. Lymphadenopathy in the adult is frequently a sign of metastatic cancer A) True B) False Question # 164 (Multiple Choice) Cats A) Tularemia B) Anthrax C) Plague
  • 14. D) Brucellosis E) Toxiplasmosis Question # 165 (Multiple Choice) Rats A) Tularemia B) Anthrax C) Plague D) Brucellosis E) Toxiplasmosis Question # 166 (Multiple Choice) Cattle A) Tularemia B) Anthrax C) Plague D) Brucellosis E) Toxiplasmosis Question # 167 (Multiple Choice) Sheep A) Tularemia B) Anthrax C) Plague D) Brucellosis E) Toxiplasmosis Question # 168 (True/False) 5. Lymphadenopathy in children is most commonly associated with infection A) True B) False Question # 169 (True/False) 6. Declawing of cats is a proper prophylaxis for CSD A) True B) False
  • 15. Question # 170 (True/False) 7. Humans may acquire CSD directly via bites from infected fleas. A) True B) False Question # 171 (Multiple Choice) 8. Using the pneumonic AEIOU TIPS as a guide for evaluation of coma, what does the letter A stand for? A) Acidosis B) Alkalosis C) Akestenia D) Amyloidosis Question # 172 (Multiple Choice) 9. Causes of seizure GENERALLY include all of the following, EXCEPT: A) hypoxia B) hypoglycemia C) infection D) pre-eclampsia Question # 173 (True/False) 10. Cat bites are more commonly the cause of CSD than scratches A) True B) False Question # 174 (Multiple Choice) A 54-year-old male patient has been thrown through a window. Glass has caused an open neck wound. You should dress this wound with: A) an occlusive dressing B) a sterile gauze dressing C) a moist gauze dressing D) butterfly bandages Question # 175 (Multiple Choice) Your patient has been exposed to an unknown chemical which has soaked his clothing. You should flush the patient with A) water B) a neutralizing solution
  • 16. C) vinegar D) a baking soda solution Question # 176 (Multiple Choice) Shock absorption and insulation are the major functions of which layer of the skin? A) dermis B) subcutaneous layer C) epidermis D) dura. Question # 177 (Multiple Choice) The major problem caused by electrical shock is often not the burn itself. The most serious problem to consider is A) shock B) hypothermia C) cardiac arrest D) brain damage Question # 178 (Multiple Choice) A 23-year-old male patient has been involved in a knife fight and has an open abdominal injury with protruding bowel. Your treatment of the injury should include A) replacing the exposed bowel B) application of the PASG for the abdominal injury C) covering the exposed bowel with a sterile dressing moistened with saline, then an occlusive dressing D) covering the bowel with dry sterile gauze prior to transport Question # 179 (Multiple Choice) Because it can become a constricting band,______ should NOT use to hold dressings in place. A) a triangular bandage B) an elastic bandage C) strips of adhesive tape D) self-adherent roller gauze Question # 180 (Multiple Choice) A 4-year-old male patient was running in the school hall. You find a pencil impaled in his upper arm. Your should
  • 17. A) apply a tourniquet to control bleeding B) remove the pencil and bandage the wound C) cut the exposed portion of the pencil off to ease bandaging and transport D) stabilize the pencil with a bulky dressing Question # 181 (Multiple Choice) A 32-year-old female patient has amputated her index finger in a table saw accident. You have bandaged her hand. The best way to transport the amputated finger is by A) wrapping the finger in gauze, placing it in a plastic bag, and keeping it cool B) placing the finger directly in a container sterile saline C) wrapping the finger well in an occlusive dressing D) placing the finger directly on dry ice Question # 182 (Multiple Choice) A patient has had his lower legs burned. The area is charred black but your patient states he has little pain. This burn would be classified as A) minor B) partial thickness C) superficial D) full thickness Question # 183 (Multiple Choice) Acceptable care for a superficial burn may include all of the following EXCEPT A) apply a burn ointment B) cool the burn with water C) keep the patient warm D) keep the burn site clean Question # 184 (Multiple Choice) When force is transmitted to the body's internal structures causing internal organs to rupture or bleed internally, or small bone fractures to occur is called a(n): A) crush injury B) concussion C) avulsion D) contusion. Question # 185 (Multiple Choice) A bruise may be an indication of internal injuries and internal bleeding. It is important for you to monitor your patient for any signs and symptoms of
  • 18. A) infection B) shock C) low blood sugar D) d. hypothermia Question # 186 (Multiple Choice) Formation of blood clots would be this mans job :-) A) White Knight Luke B) Plumber Platelet C) Erythrocyte Erythmic D) Plasma Pooler Question # 187 (Multiple Choice) If a patient is in shock, you would expect the skin to be A) cool and clammy B) warm and dry C) hot and dry D) none of the above Question # 188 (Multiple Choice) What does the "D" in "DCAP-BTLS" stand for? A) dislocations B) distal function C) dyspnea D) deformities Question # 189 (Multiple Choice) A teen male patient has a gunshot wound to the chest. Your detailed physical exam on this patient A) will begin immediately after the initial assessment B) will begin before the initial assessment C) may never begin due to the seriousness of the chest injury D) will be delayed until an ALS provider is on the scene Question # 190 (Multiple Choice) External bleeding that is a steady flow of dark red blood is from a(n):
  • 19. A) artery B) vein C) capillary D) all of the above Question # 191 (Multiple Choice) A late sign of internal bleeding is: A) bruising B) painful, swollen extremities C) signs of shock D) bright red vomitus Question # 192 (Multiple Choice) Early signs and symptoms of hypoperfusion include A) cool, clammy skin B) bradycardia C) hypertension D) constricted pupils. Question # 193 (Multiple Choice) You arrive at the scene of a bar fight. A 25-year-old man has been stabbed and is bleeding severely. The assailant is still on the scene and is holding a knife. What should you do first? A) Shoot the assailant B) Stay behind your partner C) Order the assailant to go away D) Run to the rig Question # 194 (Multiple Choice) A 34-year-old female patient has suffered a partial thickness burn to her forearm. Your treatment should include: A) applying a burn ointment B) applying ice C) puncturing blisters D) applying a dressing Question # 195 (Multiple Choice) Regarding abruptio placenta
  • 20. A) Blood loss is confined within the amniotic sack B) Internal bleeding is generally minimal. C) Blood loss may be concealed between the uterine wall and Blood loss may be concealed between the uterine wall and the placenta D) There is always excessive external vagina bleeding Question # 196 (Multiple Choice) Your patient has been crushed between a loading dock and a forklift. He presents with severe dyspnea, distended neck veins, absent breath sounds on the right side and diminished breath sounds on the left. You are able to palpate slight tracheal deviation toward the left. You should initially initial management of this patient includes immediate: A) perform a pericardiocentesis B) perform a needle decompression of the right side C) institute rapid transport D) perform transtracheal jet insufflation Question # 197 (Multiple Choice) Your homeless patient presents with bright red rectal bleeding, tachycardia and hypotension. This is most likely caused by A) bleeding ulcers B) esophageal varicies C) lower GI bleed D) upper GI bleed Question # 198 (Multiple Choice) Your patient has a third trimester bleed. You should A) perform the internal vaginal exam wearing sterile gloves B) pack the vagina cavity with gauze C) transport the patient on her left side D) perform a c-section at the scene Question # 199 (Multiple Choice) Which of the following forms of hepatitis is generally NOT chronic? A) Hepatitis A. B) Hepatitis B C) Hepatitis C D) Hepatitis D
  • 21. Question # 200 (Multiple Choice) Pale, sweaty skin, a rapid thready pulse and a delayed capillary refill time indicate: A) ICP B) hypoperfusion C) a nervous intern D) over exertion Question # 101 (Multiple Choice) You are called for an assault victim. You arrive to find a man approximately 20 years of age lying on his side in an alley. Examination reveals the patient has blood oozing from the right side of his nose. The patient's right cheek appears flattened and there is discoloration around his eyes. The patient states his cheek and nose feel numb. You suspect: Answer: (D) zygomatic fracture Question # 102 (Multiple Choice) You are on the scene of a car-pedestrian crash. A 63-year-old woman was crossing the street when she was struck by a car traveling approximately 45 miles per hour. The patient was thrown approximately 20 feet. The decision to "load-and-go" or continue stabilization at the scene will be primarily based on: Answer: (D) mechanism of injury & physical findings. Question # 103 (Multiple Choice) Select the correct statement regarding an epidural hematoma. Answer: (C) In an epidural hematoma, bleeding occurs between the cranium and the dura Question # 104 (Multiple Choice) A "Halo" test is a check for Answer: (B) CSF leaks Question # 105 (Multiple Choice) The METTAG system is one method of triaging patients. When using this system, a black tag identifies a patient who is: Answer: (A) dead or unexpected survival. Question # 106 (Multiple Choice) Responsibilities of the incident commander include: Answer: (D) rapidly assessing the situation, requesting additional resources as necessary, and developing a plan of action for management of the incident. Question # 107 (Multiple Choice) The START method of triage uses a 60-second assessment that evaluates: Answer: (C) ventilation, perfusion, mental status Question # 108 (Multiple Choice) The METTAG system is one method of triaging patients. When using this system, a green tag identifies a patient who is: Answer: (B) non-life or limb-threatening. Question # 109 (Multiple Choice) The START method of triage classifies victims as: Answer: (A) immediate, delayed, nonsalvageable/dead Question # 110 (Multiple Choice) The sector responsible for directing emergency vehicles at the scene of a major incident is the: Answer: (A) staging sector
  • 22. Question # 111 (Multiple Choice) A 27-year-old man has suffered a blow to the right side of the head. His injury may cause compression of the oculomotor cranial nerve resulting in: Answer: (A) dilation of the ipsilateral pupil. Question # 112 (Multiple Choice) You are called for an unconscious man. On arrival you find an unresponsive male, approximately 20 years of age, lying supine in a parking lot. Bystanders state he was the victim of a gang fight and has been struck in the head and shoulders with a baseball bat. Examination reveals contusions to the left temporal area and over the shoulders. There is a small amount of bleeding from the left nostril. The patient's blood pressure is 118/74, pulse 90 and regular, respiratory rate 12/min. Which of the following represents the MOST IMPORTANT intervention you can perform for this patient to help minimize cerebral edema? Answer: (A) Hyperventilate the patient. Question # 113 (Multiple Choice) The Glasgow Coma Scale evaluates: Answer: (B) eye opening, motor response, verbal response. Question # 114 (Multiple Choice) A patient who opens his eyes in response to pain, makes no verbal response, but withdraws from pain has a Glasgow Coma Score of: Answer: (C) 7 Question # 115 (Multiple Choice) Cushing's triad is associated with Answer: (D) increased intracranial pressure Question # 116 (Multiple Choice) Which of the following signs are associated with Cushing's triad? Answer: (B) increasing blood pressure, decreasing pulse rate, change in respiratory pattern Question # 117 (Multiple Choice) Select the most appropriate statement that should be used to describe a patient's level of consciousness Answer: (D) The patient withdraws from a painful stimulus Question # 118 (Multiple Choice) All of the following patients are victims involved in a motor vehicle crash. Which patient should be rapidly extricated? Answer: (C) 30-year-old male driver complaining of difficulty breathing. Unable to palpate radial pulse, carotid weak, rate 120. Respiratory rate 44 and labored Question # 119 (Multiple Choice) The MOST important sign in the evaluation of a head-injured patient is: Answer: (D) level of consciousness Question # 120 (Multiple Choice) LORAZAPAM (ATIVAN) is classified as an Answer: (A) benzodiazepine Question # 121 (Multiple Choice) The patient in the "I Feel So Ejected" case was given ATIVAN at the hospital for Answer: (C) reducing seizure activity Question # 122 (Multiple Choice) The correct adult dosage of ATIVAN is Answer: (A) 0.5 - 2.0 mg IV Question # 123 (Multiple Choice) The Mannitol dose for an adult is Answer: (A) 1.5 -2.0 g/kg IV Question # 124 (Multiple Choice) MANNITOL (OSMOTROL) is an osmotic diuretic that inhibits sodium and water absorption in the kidneys. It would most likely be used for Answer: (B) acute cerebral edema
  • 23. Question # 125 (Multiple Choice) Select the correct statement regarding an epidural hematoma. Answer: (C) In an epidural hematoma, bleeding occurs between the cranium and the dura. Question # 126 (Multiple Choice) Signs and symptoms of a basilar skull fracture may include: 1. Cerebrospinal fluid draining from the nose. 2. Ecchymosis in the mastoid region. 3. Periorbital ecchymosis. Answer: (D) d. 1, 2, 3. Question # 127 (Multiple Choice) Nitroglycerin may not be given if the patient Answer: (B) has a systolic blood pressure less than 90 Question # 128 (Multiple Choice) OPQRST are letters used to help you remember which questions to ask patients during your assessment. What does the "P" stand for? Answer: (D) provocation Question # 129 (Multiple Choice) A 45-year-old patient complains of breathing difficulty and and chest pain. He will not tolerate a non-rebreather mask. You should Answer: (A) use a nasal cannula Question # 130 (Multiple Choice) Your 40-year-old male patient complains of mild chest discomfort. As an EMT-B you should: Answer: (C) treat as if the patient has cardiac compromise. Question # 131 (Multiple Choice) Cardiac compromise commonly includes all of the following signs or symptoms EXCEPT Answer: (B) warm, dry skin. Question # 132 (Multiple Choice) Your adult patient with chest pain has a blood pressure of 92/54, a pulse of 112 and has a history of angina. You should Answer: (B) administer the patient's prescribed nitroglycerin Question # 133 (Multiple Choice) Which of the following is a shockable rythm? Answer: (B) ventricular fibrillation Question # 134 (Multiple Choice) Which of the following cardiac arrest patients should you defibrillate? Answer: (B) a 14-year-old who weighs 110 lbs Question # 135 (Multiple Answer) All of the following are effects of nitroglycerin (multi answer) (A) it dilates the blood vessels (C) it reduces the heart's work load Question # 136 (Multiple Answer) Which of the following are contraindications for the use of nitroglycerin? (multi answer) (C) The patient has also has a head injury (D) The patient took Viagra 3 hours ago Question # 137 (Multiple Choice) The cardiac conduction system disturbance that most commonly results in cardiac arrest is Answer: (B) ventricular fibrillation Question # 138 (Multiple Choice) The EMT-B requests prehospital advanced cardiac life support for the care of the cardiac arrest patient because Answer: (A) early ACLS intervention provides for higher survival rates Question # 139 (Multiple Choice) Nitroglycerin is a medication that is administered: Answer: (C) sublingually.
  • 24. Question # 140 (Multiple Choice) Why is nitroglycerin effective for patients with recurrent chest pain? Answer: (C) It dilates coronary blood vessels Question # 141 (Multiple Choice) Your 55-year-old female patient has chest pain and a blood pressure of 110/80. You give nitroglycerin. You recheck her blood pressure and are unable to obtain a reading. Her response to the nitroglycerin is called: Answer: (A) a side effect Question # 142 (Multiple Choice) Your 55-year-old female patient has chest pain and a blood pressure of 102/70. You assist with one tablet of nitroglycerin. You recheck her blood pressure and it is 80/50 and become semi- consciousness. A an EMT B, you should Answer: (D) place her in the Trendelenburg position Question # 143 (Multiple Choice) Study the picture, and then identify: Structure number 1 Answer: (B) Superior vena cava Question # 144 (Multiple Choice) Study the picture, and then identify: Structure number 5 Answer: (D) Left ventricle Question # 145 (Multiple Choice) Using the picture, which number indicates- The first chamber where blood returns to the heart Answer: (D) none of the above Question # 146 (Multiple Choice) If there are no contraindications, ___ mg of ASA should be given to the chest pain patient. Answer: (B) 324 Question # 147 (Multiple Choice) The ST segment in lead II and II is Answer: (B) elevated Question # 148 (Multiple Choice) A lateral left ventricular myocardial infarction is due to the occlusion of: Answer: (E) Left circumflex artery Question # 149 (Multiple Choice) Whichbest refers to the filling and emptying of the heart: Answer: (B) Starlings Law Question # 150 (Multiple Choice) The formation of a thrombus Answer: (D) can lead to an embolism obstructing blood flow in the artery Question # 151 (Multiple Answer) The left coronary artery supplies (multi answer) (A) LEFT VENTRICLE (B) ANTERIOR 2/3 OF THE INTERVENTRICULAR SEPTUM (C) VARIABLE PORTIONS OF THE RIGHT VENTRICLE Question # 152 (Multiple Answer) The right coronary artery supplies (multi answer) (B) SINOATRIAL NODE (C) POSTERIOR 1/3 OF THE INTERVENTRICULAR SEPTUM Question # 153 (Multiple Choice) A myocardial infarction occurs when Answer: (C) there is a blockage in a coronary artery Question # 154 (Multiple Choice) All of the following are complications of myocardial infarction except Answer: (D) Dissecting aortic aneurysm Quuestion # 155 (Multiple Choice) The reciprocal ST that matches the previous answer is seen in leads Answer: (B) I, aVL and V2
  • 25. Question # 156 (Multiple Choice) Deep and slurred S waves can be seen in leads Answer: (A) I, aVL, V5 and V6 Question # 157 (Multiple Choice) Poor R wave progression in the precordial leads along with ST elevation in V1 could suggest Answer: (B) the presence of an anterior infarction Question # 158 (Multiple Choice) Patients with poor cardiac output post nitrogylcerin administration may need to be treated with an inotropic drug, such as Answer: (C) dobutamine Question # 159 (Multiple Choice) Patients with poor cardiac output post nitrogylcerin administration may need to be treated for symptomatic bradycardia with Answer: (B) atropine Question # 160 (Multiple Choice) BONUS QUESTION: Thyroid hyperfunction leads to: Answer: (B) Marked tachycardia Question # 161 (Multiple Choice) 1. The currently recognized cause for cat scratch disease is: Answer: (B) rickettsia Question # 162 (True/False) 2. True or false, Cat Scratch disease is more common patients under the age of 21 Answer: True Question # 163 (True/False) 3. Lymphadenopathy in the adult is frequently a sign of metastatic cancer Answer: True Question # 164 (Multiple Choice) Cats Answer: (E) Toxiplasmosis Question # 165 (Multiple Choice) Rats Answer: (C) Plague Question # 166 (Multiple Choice) Cattle Answer: (D) Brucellosis Question # 167 (Multiple Choice) Sheep Answer: (B) Anthrax Question # 168 (True/False) 5. Lymphadenopathy in children is most commonly associated with infection Answer: True Question # 169 (True/False) 6. Declawing of cats is a proper prophylaxis for CSD Answer: False Question # 170 (True/False) 7. Humans may acquire CSD directly via bites from infected fleas. Answer: True Question # 171 (Multiple Choice) 8. Using the pneumonic AEIOU TIPS as a guide for evaluation of coma, what does the letter A stand for? Answer: (A) Acidosis Question # 172 (Multiple Choice) 9. Causes of seizure GENERALLY include all of the following, EXCEPT:
  • 26. Answer: (D) pre-eclampsia Question # 173 (True/False) 10. Cat bites are more commonly the cause of CSD than scratches Answer: False Question # 174 (Multiple Choice) A 54-year-old male patient has been thrown through a window. Glass has caused an open neck wound. You should dress this wound with: Answer: (A) an occlusive dressing Question # 175 (Multiple Choice) Your patient has been exposed to an unknown chemical which has soaked his clothing. You should flush the patient with Answer: (A) water Question # 176 (Multiple Choice) Shock absorption and insulation are the major functions of which layer of the skin? Answer: (B) subcutaneous layer Question # 177 (Multiple Choice) The major problem caused by electrical shock is often not the burn itself. The most serious problem to consider is Answer: (C) cardiac arrest Question # 178 (Multiple Choice) A 23-year-old male patient has been involved in a knife fight and has an open abdominal injury with protruding bowel. Your treatment of the injury should include Answer: (C) covering the exposed bowel with a sterile dressing moistened with saline, then an occlusive dressing Question # 179 (Multiple Choice) Because it can become a constricting band,______ should NOT use to hold dressings in place. Answer: (B) an elastic bandage Question # 180 (Multiple Choice) A 4-year-old male patient was running in the school hall. You find a pencil impaled in his upper arm. Your should Answer: (D) stabilize the pencil with a bulky dressing Question # 181 (Multiple Choice) A 32-year-old female patient has amputated her index finger in a table saw accident. You have bandaged her hand. The best way to transport the amputated finger is by Answer: (A) wrapping the finger in gauze, placing it in a plastic bag, and keeping it cool Question # 182 (Multiple Choice) A patient has had his lower legs burned. The area is charred black but your patient states he has little pain. This burn would be classified as Answer: (D) full thickness Question # 183 (Multiple Choice) Acceptable care for a superficial burn may include all of the following EXCEPT Answer: (A) apply a burn ointment Question # 184 (Multiple Choice) When force is transmitted to the body's internal structures causing internal organs to rupture or bleed internally, or small bone fractures to occur is called a(n): Answer: (A) crush injury Question # 185 (Multiple Choice) A bruise may be an indication of internal injuries and internal bleeding. It is important for you to monitor your patient for any signs and symptoms of Answer: (B) shock Question # 186 (Multiple Choice) Formation of blood clots would be this mans job :-) Answer: (B) Plumber Platelet
  • 27. Question # 187 (Multiple Choice) If a patient is in shock, you would expect the skin to be Answer: (A) cool and clammy Question # 188 (Multiple Choice) What does the "D" in "DCAP-BTLS" stand for? Answer: (D) deformities Question # 189 (Multiple Choice) A teen male patient has a gunshot wound to the chest. Your detailed physical exam on this patient Answer: (C) may never begin due to the seriousness of the chest injury Question # 190 (Multiple Choice) External bleeding that is a steady flow of dark red blood is from a(n): Answer: (B) vein Question # 191 (Multiple Choice) A late sign of internal bleeding is: Answer: (C) signs of shock Question # 192 (Multiple Choice) Early signs and symptoms of hypoperfusion include Answer: (A) cool, clammy skin Question # 193 (Multiple Choice) You arrive at the scene of a bar fight. A 25-year-old man has been stabbed and is bleeding severely. The assailant is still on the scene and is holding a knife. What should you do first? Answer: (D) Run to the rig Question # 194 (Multiple Choice) A 34-year-old female patient has suffered a partial thickness burn to her forearm. Your treatment should include: Answer: (D) applying a dressing Question # 195 (Multiple Choice) Regarding abruptio placenta Answer: (C) Blood loss may be concealed between the uterine wall and Blood loss may be concealed between the uterine wall and the placenta Question # 196 (Multiple Choice) Your patient has been crushed between a loading dock and a forklift. He presents with severe dyspnea, distended neck veins, absent breath sounds on the right side and diminished breath sounds on the left. You are able to palpate slight tracheal deviation toward the left. You should initially initial management of this patient includes immediate: Answer: (B) perform a needle decompression of the right side Question # 197 (Multiple Choice) Your homeless patient presents with bright red rectal bleeding, tachycardia and hypotension. This is most likely caused by Answer: (C) lower GI bleed Question # 198 (Multiple Choice) Your patient has a third trimester bleed. You should Answer: (C) transport the patient on her left side Question # 199 (Multiple Choice) Which of the following forms of hepatitis is generally NOT chronic? Answer: (A) Hepatitis A. Question # 200 (Multiple Choice) Pale, sweaty skin, a rapid thready pulse and a delayed capillary refill time indicate: Answer: (B) hypoperfusion